Derderia employment

This topic has expert replies
Master | Next Rank: 500 Posts
Posts: 377
Joined: Wed Sep 03, 2008 9:30 am
Thanked: 15 times
Followed by:2 members

Derderia employment

by schumi_gmat » Mon Oct 06, 2008 5:38 pm
With a record number of new companies starting up in Derderia, and with previously established companies adding many new jobs, a record number of new jobs were created last year in the Derderian economy. This year, previously established companies will not be adding as many new jobs overall as such companies added last year. Therefore, unless
a record number of companies start up this year, Derderia will not break its record for new jobs created.

Which of the following is an assumption on which the argument relies?

A. In a given year, new companies starting up create more new jobs on average than do previously established companies.
B. The number of people seeking employment is no larger this year than it was last year.
C. This year, the new companies starting up will not provide substantially more jobs per company than did new companies last year.
D. Previously established companies in Derderia will be less profitable this year than such companies were last year.
E. The number of jobs created in the Derderian economy last year was substantially larger than the number of jobs lost.

Can anyone explain why OA iS C
I tried to apply defender role but was not successful.

OA C

Legendary Member
Posts: 1159
Joined: Wed Apr 16, 2008 10:35 pm
Thanked: 56 times

by raunekk » Mon Oct 06, 2008 11:37 pm
imoC

conclusion:Therefore, unless a record number of companies start up this year, Derderia will not break its record for new jobs created.

C: This year, the new companies starting up will not provide substantially more jobs per company than did new companies last year.

Negation rule:This year, the new companies starting up will provide substantially more jobs per company than did new companies last year.


Thus if this is the thing,then Derderia might break its record for new jobs created.

This goes against the conclusion and the argument falls apart.

Thus C

thx

GMAT/MBA Expert

User avatar
GMAT Instructor
Posts: 3380
Joined: Mon Mar 03, 2008 1:20 am
Thanked: 2256 times
Followed by:1535 members
GMAT Score:800

by lunarpower » Wed Jul 01, 2009 3:31 pm
(c) is correct.

when you examine questions involving quantitative arguments - which this argument definitely is, even though it doesn't contain actual numbers - you should focus especially on the quantitative parts of the argument.

sure enough, in this problem, the issue is to be found in the numbers: the argument asserts that a record number of new startups must be founded. however, what's actually needed is a record number of new startup jobs. therefore, you need an assumption that solidifies the idea that, unless a record number of startups are founded, you won't get a record number of startup jobs, either.

choice c does exactly that.

--
Last edited by lunarpower on Fri May 28, 2010 3:55 am, edited 1 time in total.
Ron has been teaching various standardized tests for 20 years.

--

Pueden hacerle preguntas a Ron en castellano
Potete chiedere domande a Ron in italiano
On peut poser des questions à Ron en français
Voit esittää kysymyksiä Ron:lle myös suomeksi

--

Quand on se sent bien dans un vêtement, tout peut arriver. Un bon vêtement, c'est un passeport pour le bonheur.

Yves Saint-Laurent

--

Learn more about ron

Newbie | Next Rank: 10 Posts
Posts: 7
Joined: Tue Jun 23, 2009 8:38 pm

by trumpet1184 » Fri Jul 17, 2009 9:16 pm
Is "unless a record number of companies start up this year" also part of the conclusion? This bit throws me off, as it makes me believe that the author wants support that new startups this year could in fact break last year's record for jobs created (since existing jobs alone cannot break the record).

This reasoning has caused me to choose A, because A establishes that new companies create more jobs than existing companies. If this assumption weren't true - that is, if new companies do NOT create more jobs than existing companies do - then Derderia has no chance of breaking last year's record, which contradicts the "unless a record number of companies start up this year" statement.

I already understand the explanation for why C is correct, but I need an explanation for why A is incorrect. Could anyone please explain?

Thank you for your time.

GMAT/MBA Expert

User avatar
GMAT Instructor
Posts: 3380
Joined: Mon Mar 03, 2008 1:20 am
Thanked: 2256 times
Followed by:1535 members
GMAT Score:800

by lunarpower » Sat Jul 18, 2009 2:13 am
trumpet1184 wrote:Is "unless a record number of companies start up this year" also part of the conclusion?
absolutely yes.
This bit throws me off, as it makes me believe that the author wants support that new startups this year could in fact break last year's record for jobs created (since existing jobs alone cannot break the record).

This reasoning has caused me to choose A, because A establishes that new companies create more jobs than existing companies. If this assumption weren't true - that is, if new companies do NOT create more jobs than existing companies do - then Derderia has no chance of breaking last year's record, which contradicts the "unless a record number of companies start up this year" statement.

I already understand the explanation for why C is correct, but I need an explanation for why A is incorrect. Could anyone please explain?

Thank you for your time.
this is not a correct interpretation of the word "unless".

you are interpreting "unless X, Y" as meaning that X must happen. this is really really really bad.

if you don't see why this is bad reasoning, let's apply it to another situation.
let's say a friend tells you, "unless you live in an unsafe neighborhood, i'll spend the night at your place." this obviously doesn't mean that you are trying to prove that you live in an unsafe neighborhood.

--

here's a better way to handle "unless" statements:

you can rephrase "unless X, Y" as
"one of X or Y MUST be true."


so, the conclusion of this particular argument may be rephrased as
"either a record # of companies start up this year OR derderia will NOT break its job creation record."

--

also, don't forget that an ASSUMPTION is a statement that is 100% NECESSARY for an argument to work - i.e., if the assumption is false, then the entire argument falls apart.

if you look at choice (a) you can see that it definitely doesn't HAVE to be true, so it's wrong.
Ron has been teaching various standardized tests for 20 years.

--

Pueden hacerle preguntas a Ron en castellano
Potete chiedere domande a Ron in italiano
On peut poser des questions à Ron en français
Voit esittää kysymyksiä Ron:lle myös suomeksi

--

Quand on se sent bien dans un vêtement, tout peut arriver. Un bon vêtement, c'est un passeport pour le bonheur.

Yves Saint-Laurent

--

Learn more about ron

Legendary Member
Posts: 503
Joined: Sun Aug 09, 2009 9:53 pm
Thanked: 31 times
Followed by:2 members

by mmslf75 » Mon Dec 14, 2009 11:14 am
lunarpower wrote:
trumpet1184 wrote:Is "unless a record number of companies start up this year" also part of the conclusion?
absolutely yes.
This bit throws me off, as it makes me believe that the author wants support that new startups this year could in fact break last year's record for jobs created (since existing jobs alone cannot break the record).

This reasoning has caused me to choose A, because A establishes that new companies create more jobs than existing companies. If this assumption weren't true - that is, if new companies do NOT create more jobs than existing companies do - then Derderia has no chance of breaking last year's record, which contradicts the "unless a record number of companies start up this year" statement.

I already understand the explanation for why C is correct, but I need an explanation for why A is incorrect. Could anyone please explain?

Thank you for your time.
this is not a correct interpretation of the word "unless".

you are interpreting "unless X, Y" as meaning that X must happen. this is really really really bad.

if you don't see why this is bad reasoning, let's apply it to another situation.
let's say a friend tells you, "unless you live in an unsafe neighborhood, i'll spend the night at your place." this obviously doesn't mean that you are trying to prove that you live in an unsafe neighborhood.

--

here's a better way to handle "unless" statements:

you can rephrase "unless X, Y" as
"one of X or Y MUST be true."


so, the conclusion of this particular argument may be rephrased as
"either a record # of companies start up this year OR derderia will NOT break its job creation record."

--

also, don't forget that an ASSUMPTION is a statement that is 100% NECESSARY for an argument to work - i.e., if the assumption is false, then the entire argument falls apart.

if you look at choice (a) you can see that it definitely doesn't HAVE to be true, so it's wrong.


Ron... u rock !! ;-)

GMAT/MBA Expert

User avatar
GMAT Instructor
Posts: 3380
Joined: Mon Mar 03, 2008 1:20 am
Thanked: 2256 times
Followed by:1535 members
GMAT Score:800

by lunarpower » Tue Dec 15, 2009 4:59 am
thanks
Ron has been teaching various standardized tests for 20 years.

--

Pueden hacerle preguntas a Ron en castellano
Potete chiedere domande a Ron in italiano
On peut poser des questions à Ron en français
Voit esittää kysymyksiä Ron:lle myös suomeksi

--

Quand on se sent bien dans un vêtement, tout peut arriver. Un bon vêtement, c'est un passeport pour le bonheur.

Yves Saint-Laurent

--

Learn more about ron

Legendary Member
Posts: 549
Joined: Wed Jan 06, 2010 7:00 am
Thanked: 16 times
Followed by:3 members

by ssgmatter » Fri Apr 30, 2010 9:11 pm
@Ron....as you explained the meaning of Unless so this means that C strengthen one of the components of the conclusion which is derdia will not be able to break its record am i right
Best-
Amit

Legendary Member
Posts: 549
Joined: Wed Jan 06, 2010 7:00 am
Thanked: 16 times
Followed by:3 members

by ssgmatter » Fri Apr 30, 2010 9:22 pm
I am still confused with option A , C and B here
Best-
Amit

GMAT/MBA Expert

User avatar
GMAT Instructor
Posts: 3380
Joined: Mon Mar 03, 2008 1:20 am
Thanked: 2256 times
Followed by:1535 members
GMAT Score:800

by lunarpower » Fri May 28, 2010 3:53 am
ssgmatter wrote:@Ron....as you explained the meaning of Unless so this means that C strengthen one of the components of the conclusion which is derdia will not be able to break its record am i right
i'm not sure whether i understand the meaning of this post, but i'm a little bit worried by the presence of the word "strengthen".
it appears that you are treating "find the assumption" as if it were the same as "strengthen the conclusion".
these two categories are not at all the same; if you think that they are, you should probably check out some basic CR reading materials for the differences between the two, and then come back and post regarding questions like this one.
Ron has been teaching various standardized tests for 20 years.

--

Pueden hacerle preguntas a Ron en castellano
Potete chiedere domande a Ron in italiano
On peut poser des questions à Ron en français
Voit esittää kysymyksiä Ron:lle myös suomeksi

--

Quand on se sent bien dans un vêtement, tout peut arriver. Un bon vêtement, c'est un passeport pour le bonheur.

Yves Saint-Laurent

--

Learn more about ron

GMAT/MBA Expert

User avatar
GMAT Instructor
Posts: 3380
Joined: Mon Mar 03, 2008 1:20 am
Thanked: 2256 times
Followed by:1535 members
GMAT Score:800

by lunarpower » Fri May 28, 2010 4:00 am
ssgmatter wrote:I am still confused with option A , C and B here
here's a treatment of these choices:
A. In a given year, new companies starting up create more new jobs on average than do previously established companies.
this comparison is not relevant to the content of the argument; the argument never compares the job creation rate of new companies with that of previously established companies.

there are only two comparisons that are actually made in the passage:
1 * last year's job creation rate for previously established companies vs. this year's job creation rate for previously established companies
2 * last year's job creation rate for start-up companies vs. this year's job creation rate for start-up companies

anything that is not pertinent to these two comparisons is irrelevant here.

if you're still confused at this point, it's probably not the logic -- it's probably just the obscure topics in the problem. so here's an analogy:
* this year, the local public library made less money from donations than it did last year.
* this year, the library also made less money from fines than it did last year.
* therefore, the library will have a reduced budget this year.

in this example, if i told you "the amount of money collected from donations was greater than the amount of money collected from fines", it just wouldn't matter -- that comparison is not relevant in this example.
same thing with the problem at hand.
Ron has been teaching various standardized tests for 20 years.

--

Pueden hacerle preguntas a Ron en castellano
Potete chiedere domande a Ron in italiano
On peut poser des questions à Ron en français
Voit esittää kysymyksiä Ron:lle myös suomeksi

--

Quand on se sent bien dans un vêtement, tout peut arriver. Un bon vêtement, c'est un passeport pour le bonheur.

Yves Saint-Laurent

--

Learn more about ron

GMAT/MBA Expert

User avatar
GMAT Instructor
Posts: 3380
Joined: Mon Mar 03, 2008 1:20 am
Thanked: 2256 times
Followed by:1535 members
GMAT Score:800

by lunarpower » Fri May 28, 2010 4:01 am
B. The number of people seeking employment is no larger this year than it was last year.
this topic is irrelevant to the argument -- the argument is concerned only with the number of jobs being created, not with the number of individuals competing for those jobs once they have been created.
Ron has been teaching various standardized tests for 20 years.

--

Pueden hacerle preguntas a Ron en castellano
Potete chiedere domande a Ron in italiano
On peut poser des questions à Ron en français
Voit esittää kysymyksiä Ron:lle myös suomeksi

--

Quand on se sent bien dans un vêtement, tout peut arriver. Un bon vêtement, c'est un passeport pour le bonheur.

Yves Saint-Laurent

--

Learn more about ron

Legendary Member
Posts: 809
Joined: Wed Mar 24, 2010 10:10 pm
Thanked: 50 times
Followed by:4 members

by akhpad » Fri May 28, 2010 10:36 pm
Good explanation.

Legendary Member
Posts: 549
Joined: Wed Jan 06, 2010 7:00 am
Thanked: 16 times
Followed by:3 members

by ssgmatter » Sat May 29, 2010 9:14 pm
lunarpower wrote:
B. The number of people seeking employment is no larger this year than it was last year.
this topic is irrelevant to the argument -- the argument is concerned only with the number of jobs being created, not with the number of individuals competing for those jobs once they have been created.
Thankyou Ron!. for the expalanation.

However I don't understand how come option B talks about people competing for job???

You have already explained the interpretation of Unless in this argument however I am not able to relate C to the conclusion of the arg

When you say that Unless X Y.....means that either X or Y......so that means????.....Can you please help me showing this link with the conclusion by using option C...

Thanks!
Best-
Amit

Master | Next Rank: 500 Posts
Posts: 160
Joined: Sun May 03, 2009 1:17 am
Location: Rourkela/Hyderabad
Thanked: 4 times
Followed by:1 members

by sanp_l » Sat May 29, 2010 11:29 pm
Option C stands correct quite clearly. Its an assumption question where in a cause is stated with the effect. So if there is a missing assumption, then there is some cause which isnt stated for the effect mentioned.

The effect/Conclusion : Unless
a record number of companies start up this year, Derderia will not break its record for new jobs created.

The cause: previously established companies will not be adding as many new jobs overall as such companies added last year.

Option C provides me with another cause. This year, the new companies starting up will not provide substantially more jobs per company than did new companies last year. If this shouldn't be the case, then i won't be needing more or a record number of companies to start up new this year.

It is like this. Say last year i had 50 companies starting up with 5000 jobs, with say 100 jobs per company. Now, this year, either i need 500 companies with more than 10 jobs per company OR i need only 10 companies with 500 jobs. In either case, i get the result.

Hope it helps.
Sandy